8.2: Word Problems - UNDERLINE key words, set up your division sentence, then solve!
Ms. K's cooking club needs 2 1/4 cups of flour to make enough crepes for everyone in the class. The
club split into 6 smaller groups, each making their own batch. How much flour does each group
need?

Answers

Answer 1

Answer:

3/8

Step-by-step explanation:

hope it helps :)


Related Questions

Solve 2x + 6 < 8 or 2x + 8 > 12.

Answers

Answer:

2x + 6 < 8 = x < 1

2x + 8 > 12 = x > 2

so, for the first one it's x < 1

and for the second one it's x > 2

Step-by-step explanation:

Convert the following equation
into standard form.

y = -7x/2 - 3

Answers

Standard form: Ax + By = C

y = -7x/2 - 3
y+3 = -7x/2
2y + 6 = -7x
7x + 2y = -6

Final possible answers: 7x + 2y = -6 or
6 = -7x-2y

Answer:

7x + 2y = - 6

Step-by-step explanation:

the equation of a line in standard form is

Ax + By = C ( A is a positive integer and B, C are integers )

given

y = - [tex]\frac{7}{2}[/tex] x - 3 ( multiply through by 2 to clear the fraction )

2y = - 7x - 6 ( add 7x to both sides )

7x + 2y = - 6 ← in standard form

Drag the tiles to the correct boxes to complete the pairs. Not all tiles will be used.
Solve the equations for x and match the solutions.

Answers

The solution to each of the equations are matched together as:

1. 4 = 6/a*x + 5 → x = -a/6

2. 7 + 2ax = 13 → x = 3/a

3. -ax - 20 = -14 → x = -6/a

How to Solve an Equation?

To solve an equation, apply the properties of equality in order to isolate the variable solved for to one side of the equation, which gives us the solution to the problem.

1. 4 = 6/a*x + 5

4 = 6x/a + 5

Combine like terms

-6x/a = -4 + 5

-6x/a = 1

-6x/a(a) = 1(a) [multiplication property of equality]

-6x = a

-6x/-6 = a/-6 [division property of equality]

x = -a/6

2. 7 + 2ax = 13

2ax = 13 - 7

2ax = 6

Divide both sides of the equation by 2a

x = 6/2a

x = 3/a

3. -ax - 20 = -14

-ax = -14 + 20

-ax = 6

x = 6/-a

x = -6/a

Therefore, the solutions are:

1. x = -a/6

2. x = 3/a

3. x = -6/a

Learn more about equations on:

https://brainly.com/question/25678139

#SPJ1

Word problems ( put solution )

1. John mixed 10.84 liters of honey water with 12.2 liters of
lemon juice to make a drink. Then he poured the mixture
into 0.75-liter bottles.
a) How many bottles could he fill completely with the mixture?
b) How many liters of the mixture were left

2.Mr. Alai bought a digital music player at $76.55 and a
memory card at $10.95 before sales tax. If the sales tax
was 10%, find the total amount he had to pay.

Answers

The number of bottles that could be completely filled is: 30 bottles. Additionally, he has 0.72 liters of drink left. On the other hand, the total price of the music player and memory card is $96.25 (with tax).

How to solve problem 1?

To solve problem 1 we need to consider how much drink John got after mixing the honey water with the lemon juice.

10.84 + 12.2 = 23.04

Subsequently, we divide the total juice in quantities of 0.75 to obtain the number of bottles that we can fill, as shown below:

32.04 / 0.75 = 30.72

According to the above, 30 bottles of 0.75 liters can be filled and there would be a remaining 0.72 liters.

How to solve problem 2?

To solve problem 2 we must add the value with tax of the two products

$76.55 + $10.95 = $87.5

Later, to find 10% of this value we must divide it by 100 and multiply the result by 10 as shown below:

$87.5 / 100 = 0.8750.875 * 10 = $8.75

Finally, we must add this value to the total value without tax ($87.5) to obtain the total value with taxes

$87.5 + $8.75 = $96.25

Learn more about mathematics in: https://brainly.com/question/15209879

#SPJ1

Which equations represent the line that is parallel to 3x - 4y = 7 and passes through the point (-4, -2)? Select two
options.
Oy=-¾x+1
V 3x - 4y = -4
O 4x - 3y = -3
Oy-2=-¾(x-4)
Oy+2=¾(x+4)
Mark this and return
Save and Exit
Next
Submit

Answers

Answer:

Step-by-step explanation:

5

The equation of line passes through the point (-4, -2) will be;

⇒ 3x - 4y = - 4

What is Equation of line?

The equation of line in point-slope form passing through the points

(x₁ , y₁) and (x₂, y₂) with slope m is defined as;

⇒ y - y₁ = m (x - x₁)

Where, m = (y₂ - y₁) / (x₂ - x₁)

Given that;

The point on the line are (-4, -2).

And, The parallel line is,

⇒ 3x - 4y = 7

⇒ y = 3/4x - 7/4

Now,

Since, The equation of line passes through the point (- 4, -2).

So, We need to find the slope of the line.

Since, The slope of the parallel line is same.

Hence, Slope of the line is,

⇒ y = 3/4x - 7/4

m = dy/dx = 3/4

m = 3/4

Thus, The equation of line with slope 3/4 is,

⇒ y - (-2) = 3/4 (x - (-4))

⇒ y + 2 = 3/4 (x + 4)

⇒ y + 2 = 3/4x + 3

⇒ y = 3/4x + 3 - 2

⇒ y = 3/4x + 1

⇒ 4y = 3x + 4

⇒ 3x - 4y = - 4

Therefore, The equation of line passes through the point (-4, -2) will be;

⇒ 3x - 4y = - 4

Learn more about the equation of line visit:

https://brainly.com/question/18831322

#SPJ5

Please help need answer now!!!!!! Pleaseee

Keaton knows that the slope of a certain line is m = -8/5. Does he have enough information to write the equation of the line? Explain. A. Yes the y intercept of this equation must be (0,0) so you can write the equation with only the slope value B. yes if you know the value of the slope, then you can guess the location of the y intercept and write the equation C. No, to write the equation of a line do you need to know both the Slope and one ordered pair that lies on the line D. No you don’t need to know the slope of a line to write its equation 

Answers

Answer:

C

Step-by-step explanation:

the equation of a line in slope- intercept form is

y = mx + c ( m is the slope and c the y- intercept )

the equation of a line in point- slope form is

y - b = m(x - a)

where m is the slope and (a, b ) a point on the line

To obtain either equation we require to know both the slope and one ordered pair that lies on the line.

URGENT!! ILL GIVE
BRAINLIEST!!!! AND 100 POINTS!!!!!

Answers

Answers in bold

Type of angle pair is same-side interior angles. These angles are supplementary so x = 46 degrees.

=================================================

Explanation:

The angles shown are known as "same-side interior" or "consecutive interior". They are interior because they are inside the parallel train tracks. They are considered "same-side" because both are below the transversal line.

If the train tracks are parallel, then those same-side interior angles are supplementary. This means they add to 180 degrees.

x+134 = 180

x+134-134 = 180-134

x = 46

y= 10
_
X
Is linear with or no linear

Answers

Answer:

Linear

Step-by-step explanation:

because it is in the form y=mx+c if rearranged

What is an equation of the line that passes through the points (4, 8)(4,8) and (4, 3)(4,3)?

Answers

Answer:

x=4

Step-by-step explanation:

x=4 would go through 4,3 and 4,8 as 4 is a vertical line

gabriel is bored and starts writing his name over and over again on his paper. what is the 1000th letter?

Answers

Answer:

'e'

Step-by-step explanation:

number of letter in his name: 7

number of times his name is written at the 1000th letter

= 1000/7 = 142 R 6

since there is a remainder of 6 letters after the previous complete set of his name, therefore the 1000th letter is the 6th letter of his name, and that is e

What are th roots of 3x^2+10=4x

Answers

The roots of the Quadratic Equation are

(2 + i√26)/3 and (2 - i√26)/3

Given, a quadratic equation i.e. 3x² + 10 = 4x

On simplifying, we get

3x² - 4x + 10 = 0

while checking the discriminant of the given quadratic equation, we get

D = b² - 4ac

D = (-4)² - 4(3)(10)

D = 16 -120

D = -104

i.e. D < 0.

So, the quadratic equation has imaginary roots

On applying quadratic formula, we get

x = (4 ± √(-104))/6

x = (4 ± i2√26)/6

x = (2 ± i√26)/3

Hence, the roots of the Quadratic Equation are

(2 + i√26)/3 and (2 - i√26)/3

Learn more about Quadratic Equations here https://brainly.com/question/1214333

#SPJ1

One plane can travel 10 miles per hour faster than another. One of them goes 135 miles in the same time it takes the other to go 130 miles. What are their speeds?

Answers

The speed of the slower plane is 260 mph and the speed of the faster plane is 270 mph.

What is the distance?

Distance is defined as the product of speed and time.

Let the speed of the slower plane would be s

then (s+10) = speed of the faster

So time equation; time = dist/speed

130 / s = 135 / (s+10)

130(s+10) = 135s

130s + 1300 = 135s

135s - 130s = 1300

5s = 1300

s = 1300 / 5

s = 260

Since s = 260 mph is the speed of the slower plane

So, 270 mph is the speed of the faster.

Thus, the speed of the slower plane is 260 mph and the speed of the faster plane is 270 mph.

Learn more about distance here:

brainly.com/question/13269893

#SPJ1

i need help QUICK
Evaluate.

−[4.1−3(2.2) ]
___________ +2
(−0.2)^2

What is the value of the expression?

Enter your answer as a decimal in the box.

Answers

The value of the given expression in the question, -[4.1-3(2.2)] / (0.2)² + 2       is 64.5.

What do you mean by expression?

An expression in mathematics is made up of numbers, variables, and functions (such as addition, subtraction, multiplication or division etc.)

You can think of expressions as being similar to phrases. A phrase in language may contain an action on its own, but it does not constitute a complete sentence.

-[4.1-3(2.2)] / (0.2)² + 2

Using the BODMAS rule to simplify the given expression.

Given expression can be written as:

= -[4.1-6.6] / 0.04 + 2

= -[-2.5]/0.04+2

=2.5/0.04+2

=62.5+2

=64.5

Therefore, 64.5 is answer for expression  -[4.1-3(2.2)]/(0.2)^2+2.

To know more about expression, go to link

https://brainly.com/question/723406

#SPJ9

please help!
thank u.

Answers

a) In midday there is no change in the height of the water.

b) At 19.00 hours the height of water will decrease 1.5

Given,

The height of the water = h

Equation, h = 3sin(30t)°

t is the time in hours

a) We have to find the height of water in midday;

Midday = 12 noon

Now,

h = 3sin(30t)²

t = 12

Then,

h = 3sin(30 x 12)°

h = 3sin360°

h = 0

That is,

In midday there is no change in the height of the water.

b) We have to find the height of the water at 19.00

Now,

h = 3sin(30t)²

t = 19

Then,

h = 3sin(30 x 19)°

h = 3sin570°

h = -1.5°

That is,

At 19.00 hours the height of water will decrease 1.5

Learn more about height of water here;

https://brainly.com/question/13594154

#SPJ1

Type the correct answer in the box. Use numerals instead of words. For this item, a non-integer answer should be entered as a fraction using / as the fraction bar. Simplify the numerical expression. The expression has a value equal to


[tex] \frac{2}3 \: \div {2}^{4} + ( \frac{3}{4} + \frac{1}{6} ) \: \div \frac{1}{3} [/tex]

Answers

After solving thie expression using BODMAS rule we get the result equal to  [tex]\frac{67}{24}[/tex].

What is BODMAS?

To solve arithmetic expressions of several operators, the BODMAS rule is a mathematics is a technique. We will solve an equation with several operators with the use of BODMAS. It provides the decoding of the mathematical phrase in the proper sequence.

B= Bracket

O= Of

D= Divide

M= Multiply

A= Addition

S= Subtraction

To solve this numerical expression,

[tex]\frac{2}{3}[/tex] ÷ 2⁴ + ([tex]\frac{3}{4}[/tex] + [tex]\frac{1}{6}[/tex]) ÷ [tex]\frac{1}{3}[/tex]

Use BODMAS method

[tex]\frac{2}{3}[/tex] ÷ 16 + ([tex]\frac{9+2}{12}[/tex]) ÷ [tex]\frac{1}{3}[/tex]

[tex]\frac{2}{3}[/tex] ÷ 16 + ([tex]\frac{11}{12}[/tex]) ÷ [tex]\frac{1}{3}[/tex]

[tex]\frac{2}{3}[/tex]  × [tex]\frac{1}{16}[/tex] + ([tex]\frac{11}{12}[/tex]) × 3

[tex]\frac{1}{24}[/tex] + [tex]\frac{33}{12}[/tex]

[tex]\frac{67}{24}[/tex]

The expression has a value equal to [tex]\frac{67}{24}[/tex].

To know more about  expression , visit:

https://brainly.com/question/14083225

#SPJ1

a jet plane travels 1,290 in 3 hours Assuming its speed is constant, how long will it take to travel 5,160 Miles?

Answers

The time it takes is 12 hours to travel 5,160 Miles.


The time is take can be calculated as follows?


The first is calculate the speed of jet plane in miles/hour (mph)

= 1,290 / 3

= 430 mph

Next is to calculate the time is take to travel 5,160 by dividing the distance by the speed

= 5,160 / 430

= 12 hours

Thus, the time it takes is 12 hours.

Learn more about speed here:

brainly.com/question/7359669

#SPJ1

On a coordinate plane, the y-axis is labeled imaginary and the x-axis is labeled real. Point Z 1 is (3, 1), Z 2 is (negative 2, negative 5). Point A is at (1, negative 4), B is (5, 5), C is (negative 3, 1), and D is (4, negative 2). A dotted line goes from z 1 to (0, 0), and then down to point z 2. Which point represents z1 + z2?

Answers

The sum z1 + z2 gives the point with coordinates (1, - 4), so option A is the correct one.

Which point represents z1 + z2?

We know that these two points have the coordinates:

z1 = (3, 1)

z2 = (-2, - 5)

And we want to find the addition of the two points.

To add them, we just need to add the correspondent coordinates:

(3, 1) + (-2, -5) = (3 - 2, 1 - 5) = (1, - 4)

That point is the sum of z1 and z2, and we know that it is the point A, so point A is the correct option.

Learn more about coordinates:

https://brainly.com/question/11337174

#SPJ1

round to the nearest ten thousand
a.12,456
b.60,792
c.93,245​

Answers

As per the rule of round off, the nearest ten thousand to the numbers are

a.12,456 = 10,000

b.60,792 = 60,000

c.93,245​ = 90,000

Round off

Round off defines a change a number into an approximation having fewer significant digits.

Given,

Here we need to round off the following numbers to the nearest ten thousand.

a.12,456

b.60,792

c.93,245​

a. 12,456

If you rounded to the nearest ten thousands place. The 1 in the ten thousands place rounds down to 1, or stays the same, because the digit to the right in the thousands place is 2.

If the digit to the right is less than 5 we round toward 0.

Therefore, 12,456 was rounded down toward zero to 10,000

b. 60,792

If you rounded to the nearest ten thousands place. The 6 in the ten thousands place rounds down to 6, or stays the same, because the digit to the right in the thousands place is 0.

So, if the digit to the right is less than 5 we round toward 0.

Then 60,792 was rounded down toward zero to 60,000

c. 93, 245

Similar to the previous one, if rounded to the nearest ten thousands place. The 9 in the ten thousands place rounds down to 9, or stays the same, because the digit to the right in the thousands place is 3.

When the digit to the right is less than 5 we round toward 0.

93245 was rounded down toward zero to 90,000

To know more about Round off here.

https://brainly.com/question/13391706

#SPJ1

Please help will mark Brainly

Answers

Answer:

B

Step-by-step explanation:

I did it, I know, can I have brainliest please?

Country S has 12,345,000 residents who tested positive for COVID-19. What is the largest place value of the numeric part of this quantity?

Answers

The largest place value of the numeric part of this quantity is 10 million.

What exactly is a place value in mathematics?

Our entire number system is built around place value. This is the system in which the value of a number is determined by the position of its digits. The numbers 42,316 and 61,432 differ because the digits are in different positions.

Every digit in a number has a place value in mathematics. Place value is the value represented by a digit in a number based on its position in the number. For example, 7 hundreds or 700 is the place value of 7 in 3,743. The place value of 7 in 7,432 is, however, 7 thousands or 7,000.

In the number, 12,345,000, the highest place value is 1 and it denotes 10 million.

Learn more about place value on:

https://brainly.com/question/1654524

#SPJ1

Kim spent $65 dollars on a new computer but she gave away $10 dollars to her friend Samantha.How many dollars does Kim have left to buy her new computer.

Answers

Answer:

Step-by-step explanation:Le quedo +10$

Ya que

-65 y le regalaron +10

If d = 2, evaluate 6d.

Answers

Answer:

12

Step-by-step explanation:

1) Substitute d into the question

We can see that the question says that is d is 2 what would 6d be.

Another way of writing 6d would be 6 x d

If we put d into the equation we would be left with 6 x 2

2) Solve

6 x 2 = 12

Hope this helps, have a great day!!

Answer:

12

Step-by-step explanation:

6d=6(2)

6*2=12

HOPE IT HELPS

a fair die was thrown once what is the probability of getting either even or 6​

Answers

Answer:

1/2 or 1/6

Step-by-step explanation:

a fair die has 6 possible outcomes:

1,2,3,4,5, or 6

probability of getting an even number is:

3/6 = 1/2 {as there are three possible even outcomes}

probability of getting 6 is:

1/6

Help me please!!!!!!!!!!!!

Answers

Answer:

1. 2,772

2.1,071

3.4,641

4. 36,610

5. 4,096

6. 36,270

7. 2,403

8. 3,320

Determine which of the lines if any are parallel or perpendicular

Answers

Answer:

Parallel: Lines a and b

Perpendicular: none

Step-by-step explanation:

Both lines a and b have a slope of 1/3.

This means they are parallel, so they'll never intersect.

Perpendicular lines form a right angle, and none seem to do see here.

A family has a $42,000 annual salary. Can they afford an $85,000 house with a $65,000 mortgage requiring payments of $720 per month, including principal, interest, taxes, and insurance?

Answers

Yes! they can afford an $85,000 house with a $65,000 mortgage requiring payments of $720 per month.

What is in a mortgage?

Principal, interest, taxes, and insurance make up the majority of mortgage payments. Your outstanding loan balance is reduced by the amount of the principal part. Borrowing money has a cost, which is interest.

Given data:

Annual Salary =$42,000

Cost =$85,000

Mortgage = $65000

Monthly Payment = $720

First Criterian : - Never Spend more than 2.5 times of your Annual Income.

2.5 * 42,000

= 105,000

it can be afforded.

Second Criterian : - Monthly Payment should not exceed Weekly Income

Monthly Payment = 720

Weekly earnings = 42000 / 52 weeks = 807

it can afforded.

Third Criterian : - monthly payment should not exceed 28% of monthly earnings.

Monthly payment = 720

Monthly earnings = 42,000 / 12 = 3500

28% * 3500

= 980

it can be afforded.

To learn more about the mortgage from the given link

https://brainly.com/question/1318711

#SPJ1

Based on the graph above, determine the amplitude, midline, and period of the function
Amplitude:?
Period:?
Midline: y=?

Answers

The amplitude, midline and period of the function are 3 units, 4units and 5 units respectively

How to determine the amplitude, midline, and period of the function based on the graph?

The amplitude is half the full height of the periodic function. That is:

Amplitude = (maximum - minimum)/2

Check the image attached, the maximum and minimum are labeled max and min respectively

Amplitude = (-1 - (-7))/2  = 6/2 = 3 units

The midline is the middle of the periodic graph or halfway between the minimum and maximum of the graph

Midline = (maximum + minimum)/2

Midline = (-1 + (-7))/2 = -8/2 = -4 units

The period is the distance between two peaks or (maximums) on the graph.

Period = 1-(-4) = 5 units

Therefore, the amplitude, midline and period are 3 units, 4units and 5 units respectively

Learn more about period function on:

https://brainly.com/question/23427826

#SPJ1

A box of candy had 8 cherry pieces for every 3 lemon pieces. If the box had 30 lemon pieces how many cherry pieces would there be

Answers

Answer:

240

Step-by-step explanation:

30 X 8

If T is the midpoint of RS and V lies between R and T, which statement must be true?

Answers

T is the midpoint of RS and V lies between Rand T is RV+VT =TS

Explain about the midpoint?

The midpoint of a line segment is known as the midpoint in geometry. It is the centroid of the segment and of the endpoints, and it is equally distant from both of them. It cuts the section in half.

Divide the measurement of the distance between the two end locations by 2. The middle of that line is located at this separation from either end. Alternately, combine the two endpoint x coordinates and divide by 2. similarly for the y coordinates.

The midpoint rule substitutes the midpoints, mi, of each subinterval for xi in a Remain sum with subintervals of equal width to estimate definite integrals.

T is the midpoint of RS so RT =TS

To learn more about midpoint refer to:

https://brainly.com/question/18315903

#SPJ13

T is the midpoint of RS and V lies between Rand T is RV+VT =TS

Explain about the midpoint?In geometry, a line segment's midpoint is referred to as the midpoint. It is both the segment's and the endpoints' centroid, and it is equally far away from each. The portion is halved by it.The distance between the two end points should be divided by two. This distance from either end is where the centre of that line is situated. You may also add the two endpoint x coordinates and divide the result by 2. the same is true for the y coordinates.The midpoint rule uses subintervals of equal width to replace the midpoints, mi, of each subinterval in a Remain sum for xi in order to estimate definite integrals.

RT = TS because T is the midpoint of RS.

To learn more about midpoint refer to:

https://brainly.com/question/18315903

#SPJ13

Answer question in picture

Answers

The given graph can be expressed as an equation of f(x) = -2cos(-x).

What is a graph of a trigonometric function?

In these trigonometry graphs, the x-axis values of the angles are in radians, and the y-axis value of the function at each given angle is taken as f(x).

The given graph is of a negative angle of a negative cosine.

So if we draw the graph of the negative angle of a negative cosine, the graph will pass through the origin, but

If we multiply 2 from the graph of the negative angle of a negative cosine then we will get the same graph as shown in the given figure.

So the equation would be f(x) = -2cos(-x).

Hence, the equation of the graph will be f(x) =  -2cos(-x).

To learn more about trigonometric functions, visit:

brainly.com/question/24349828

#SPJ1

Other Questions
help me with this please What impact do you think gpa and extracurriculars might have on the college options available to you and how much you will have to pay?. Explain the reason of the instability of the - helix caused by: Proline, Glycine,Valine, & Tryptophan Angelo works as the frozen food manager in a large grocery store. When his stock of two-pound bags of frozen shrimp gets down to two cases, he emails the warehouse to send ten cases to restock. The type of decision angelo is making about restocking shrimp is a. What is the solution to 2|2.2x 3.3| = 6.6?x = 3x = 3x = 3 or x = 0x = 0 or x = 3 In onion root tips, a new cell wall can be seen as it forms between two daughter cells during which phase of mitosis?. If the sprinter from the previous problem accelerates at that rate for 20. 00 m and then maintains that velocity for the remainder of a 100. 00-m dash, what will her time be for the race?. hornberger, inc. recently paid a dividend of $2.00 per share. the next dividend is expected to be $2.06 per share. hornberger has a return on equity of 12.20%. what percentage of its earnings does hornberger plow back into the firm? (round your answer to 2 decimal places.) The following is a DNA nucleotide sequence. Write the sequence of nucleotides that would pair up with this sequence: GCA TCT CCC CGA motor pathways for the control of skeletal muscles, like the diaphragm and external intercostal muscles, exhibit what kind of neuronal organization? I need answer by today, please. And Thanks! The renowned sixteenth-century musical composer johann sebastian bach was also an outstanding soprano until he was about sixteen. The change in his voice was a characteristic of For science class, Joan watches a bird as it moves upand down in a tree over time. You could make agraph to represent the relationship between thebird's height off the ground and time. If you made agraph, what variable would you put on each axis?Explain your answers.Independent (x-axis) variable:Dependent (y-axis) variable:1 Which of the following were part of the Treaty of Paris 1783? Select ALL that apply.Select 3 correct answer(s)The US would have to purchase the western lands from the British for 7 million dollarsThe British would continue to have economic control over the United StatesThe British were to fully withdraw all troops from US landsThe British would cede all lands east of the Mississippi River and North to Canada to the United States except for Florida, which would be given back to SpainThe United States would gain independence Graph the equation using the point and the slope. y-5=1/3(x-1) If you are charged with selling/providing/delivering alcoholic beverages to an intoxicated person, the penalty is a class b misdemeanor. For the first offense, the maximum monetary penalty could be. you are grilling burgers for a party that will have 120 guests. you believe that a guest might need 0, 1, or 2 burgers with probabilities 0.2, 0.7, and 0.1, respectively. you assume that the number of burgers each guest needs is independent from other guests. how many burgers should you make so that you are 95% sure there will be enough? Solve the system of equations 2x + 5y = -9 and 3x + 4y = 4 by combining theequations.try(2x(3x+ 5y =-9)+ 4y = 4)2x +5y = -93x +4y= 40x0y= no matter the size of the business, finance is a critical activity for multiple choice profit-seeking and nonprofit organizations. nonprofit organizations, but not for profit-seeking businesses. accountants, but not for financial managers. profit-seeking, but not for nonprofit organizations. A video treating company offers two monthly plans. Plan A: 3$ per video viewed, plus a flat rate of 8$ per month.Plan B: 5$ per video viewed and no additional flat rate. Write an inequality to determine when the cost of viewing n videos using Plan A is less then the cost of viewing n videos using plan B. Inequality: Plan A is less expensive when: